Spring SHM Homework: Solving for Displacement of a Suspended Point Mass

In summary, the discussion is about finding the equation of displacement of a point mass suspended by a massless spring with a constant of 2000 N/m. The point mass is released at an elongation of 15 cm and the equation of displacement is given as x(t)=15cos(10t). The values of ω and x are also determined. The unknown values of phi and amplitude are then calculated using the equation for displacement.
  • #1
Suyash Singh
168
1

Homework Statement


A point mass m= 20 kg, is suspended by a
massless spring of constant 2000 N/m. The
point mass is released when elongation in
the spring is 15 cm. The equation of
displacement of particle as function of time
is : (Take g = 10 m/s2)

Homework Equations



A is amplitude
w is omega
t is time
k is spring constant

The Attempt at a Solution


PE= 1/2 m A^2 w^2 cos^2(wt+ phi)
PE=1/2Kx^2
equating these two equations doesn't do anything so how do i proceed?
 
Physics news on Phys.org
  • #2
Suyash Singh said:
doesnt do anything
Can you elaborate ? What is ##\omega## ? What is PE in the first equation ?
 
  • #3
Suyash Singh said:
equating these two equations doesn't do anything
It does if you consider specific values of x and t, i.e. the boundary conditions.
You will also need more equations since you have three unknowns to pin down.
 
  • Like
Likes BvU
  • #4
s
BvU said:
Can you elaborate ? What is ##\omega## ? What is PE in the first equation ?
omega is angular velocity
PE is potential enrergy
 
  • #5
haruspex said:
It does if you consider specific values of x and t, i.e. the boundary conditions.
You will also need more equations since you have three unknowns to pin down.
i really don't know what to do
 
  • #6
So you want to find the equation of motion? Don't forget that the mechanical energy is being affected by gravity as well as the spring. So PEgravity=mgx

You only used the kinetic energy and spring potential energy.
 
  • Like
Likes Suyash Singh
  • #7
Suyash Singh said:
i really don't know what to do
What do you know about x, ##\dot x## and P when t=0?
 
  • Like
Likes Suyash Singh
  • #8
haruspex said:
What do you know about x, ##\dot x## and P when t=0?
when t=0 x=15
dont know what is x dot(if it is omega then i don't know how to find it)
PE=1/2 2000 (15/100)^2
=1000 x 225/10000
22.5 joules
 
  • #9
Send BoBs said:
So you want to find the equation of motion? Don't forget that the mechanical energy is being affected by gravity as well as the spring. So PEgravity=mgx

You only used the kinetic energy and spring potential energy.
but i don't know the distance from ground so PEgravity=??
 
  • #10
Here is what I would do. Just keep in mind that I only have the knowledge of a first year physics student

Particle executes SHM ∴
x(t)=xmcos(ωt-θ)
v(t)=-ωxmsin(ωt-θ)
a(t)=-ω2xmcos(ωt-θ)

F=ma => -1/2kxm2=m(-ω2xm) => ω=√k/m=10rad/s

x̶[̶S̶U̶B̶]̶m̶[̶/̶S̶U̶B̶]̶=̶0̶.̶0̶1̶5̶m̶, ω=10rad/s

v̶(̶0̶)̶/̶x̶(̶0̶)̶=̶-̶ω̶x̶[̶S̶U̶B̶]̶m̶[̶/̶S̶U̶B̶]̶s̶i̶n̶θ̶/̶x̶[̶S̶U̶B̶]̶m̶[̶/̶S̶U̶B̶]̶c̶o̶s̶θ̶=̶-̶ω̶t̶a̶n̶θ̶ ̶=̶>̶ ̶t̶a̶n̶θ̶=̶-̶(̶(̶v̶(̶0̶)̶/̶x̶(̶0̶)̶)̶/̶ω̶)̶ ̶=̶>̶ ̶θ̶=̶t̶a̶n̶[̶S̶U̶P̶]̶-̶1̶[̶/̶S̶U̶P̶]̶-̶(̶(̶v̶(̶0̶)̶/̶x̶(̶0̶)̶)̶/̶ω̶)̶ ̶s̶i̶n̶c̶e̶ ̶x̶(̶0̶)̶ ̶a̶n̶d̶ ̶v̶(̶0̶)̶ ̶a̶r̶e̶ ̶u̶n̶k̶n̶o̶w̶n̶ ̶t̶h̶i̶s̶ ̶m̶e̶t̶h̶o̶d̶ ̶f̶a̶i̶l̶s̶.̶ ̶ ̶∴̶x̶(̶t̶)̶=̶0̶.̶0̶1̶5̶m̶c̶o̶s̶(̶1̶0̶(̶r̶a̶d̶/̶s̶)̶t̶-̶θ̶)̶ ̶w̶h̶e̶r̶e̶ ̶o̶n̶l̶y̶ ̶θ̶ ̶i̶s̶ ̶l̶e̶f̶t̶ ̶u̶n̶k̶n̶o̶w̶n̶.̶ ̶T̶h̶i̶s̶ ̶i̶s̶ ̶a̶l̶l̶ ̶a̶s̶s̶u̶m̶i̶n̶g̶ ̶t̶h̶a̶t̶ ̶x̶[̶S̶U̶B̶]̶m̶[̶/̶S̶U̶B̶]̶=̶1̶5̶c̶m̶

There's a solution for this on https://www.hashlearn.com/questions/study_levels/class-xi/exams/engineering-entrance/subjects/physics/topics/oscillations-and-waves/challenges/point-mass-20-kg-suspended-massless-spring-constant-2000-nm-point-mass-released-elongat-aa4b9c6 but need to pay to see. If you're really desperate... I still wouldn't pay for that kind of information (should totally be free).

Also if you are using the mechanical energy the height for mgh would just be the distance x that is given when the particle is back at 0 potential spring energy and 0 when its first released.
 
Last edited:
  • Like
Likes Suyash Singh
  • #11
Suyash Singh said:
when t=0 x=15
First, need to define x. You need to specify where 0 is (bearing in mind that you have to match it with an SHM expression) and whether up is positive or negative.
Suyash Singh said:
dont know what is x dot
If x is displacement, what is dx/dt? What is its value when the mass is released?
 
  • Like
Likes Suyash Singh
  • #12
Send BoBs said:
Here is what I would do. Just keep in mind that I only have the knowledge of a first year physics student

Particle executes SHM ∴
x(t)=xmcos(ωt-θ)
v(t)=-ωxmsin(ωt-θ)
a(t)=-ω2xmcos(ωt-θ)

F=ma => -1/2kxm2=m(-ω2xm) => ω=√k/m=10rad/s

xm=0.015m, ω=10rad/s

v̶(̶0̶)̶/̶x̶(̶0̶)̶=̶-̶ω̶x̶[̶S̶U̶B̶]̶m̶[̶/̶S̶U̶B̶]̶s̶i̶n̶θ̶/̶x̶[̶S̶U̶B̶]̶m̶[̶/̶S̶U̶B̶]̶c̶o̶s̶θ̶=̶-̶ω̶t̶a̶n̶θ̶ ̶=̶>̶ ̶t̶a̶n̶θ̶=̶-̶(̶(̶v̶(̶0̶)̶/̶x̶(̶0̶)̶)̶/̶ω̶)̶ ̶=̶>̶ ̶θ̶=̶t̶a̶n̶[̶S̶U̶P̶]̶-̶1̶[̶/̶S̶U̶P̶]̶-̶(̶(̶v̶(̶0̶)̶/̶x̶(̶0̶)̶)̶/̶ω̶)̶ ̶s̶i̶n̶c̶e̶ ̶x̶(̶0̶)̶ ̶a̶n̶d̶ ̶v̶(̶0̶)̶ ̶a̶r̶e̶ ̶u̶n̶k̶n̶o̶w̶n̶ ̶t̶h̶i̶s̶ ̶m̶e̶t̶h̶o̶d̶ ̶f̶a̶i̶l̶s̶.̶ ̶ ̶∴̶x̶(̶t̶)̶=̶0̶.̶0̶1̶5̶m̶c̶o̶s̶(̶1̶0̶(̶r̶a̶d̶/̶s̶)̶t̶-̶θ̶)̶ ̶w̶h̶e̶r̶e̶ ̶o̶n̶l̶y̶ ̶θ̶ ̶i̶s̶ ̶l̶e̶f̶t̶ ̶u̶n̶k̶n̶o̶w̶n̶.̶ ̶T̶h̶i̶s̶ ̶i̶s̶ ̶a̶l̶l̶ ̶a̶s̶s̶u̶m̶i̶n̶g̶ ̶t̶h̶a̶t̶ ̶x̶[̶S̶U̶B̶]̶m̶[̶/̶S̶U̶B̶]̶=̶1̶5̶c̶m̶

There's a solution for this on https://www.hashlearn.com/questions/study_levels/class-xi/exams/engineering-entrance/subjects/physics/topics/oscillations-and-waves/challenges/point-mass-20-kg-suspended-massless-spring-constant-2000-nm-point-mass-released-elongat-aa4b9c6 but need to pay to see. If you're really desperate... I still wouldn't pay for that kind of information (should totally be free).

Also if you are using the mechanical energy the height for mgh would just be the distance x that is given when the particle is back at 0 potential spring energy and 0 when its first released.
Its ok cause its a school problem
ok so omega, x are known now
i need phi and amplitude
x(t)=xsin(wt+ phi)
at t=0
15=15sin(phi)
phi=90 degrees
x(t)=15sin(10t+90)=15cos(10t)
similarly for others
v(t)=-150sin(10t)
a(t)=-1500cos(10t)
 
  • #13
haruspex said:
First, need to define x. You need to specify where 0 is (bearing in mind that you have to match it with an SHM expression) and whether up is positive or negative.

If x is displacement, what is dx/dt? What is its value when the mass is released?
x is displacement
x=0 is the condition when mass is not present on spring
down is positive cause its more convienient this way here
dx/dt is velocity
mass released then velocity=0 m/s cause of the above equation which i wrote in post #12
 
  • #14
Suyash Singh said:
x=0 is the condition when mass is not present on spring
For the standard SHM equation you need the displacement from the equilibrium position. Otherwise you will need to add a constant term.
So think again about the amplitude.
 
  • Like
Likes Suyash Singh
  • #15
haruspex said:
For the standard SHM equation you need the displacement from the equilibrium position. Otherwise you will need to add a constant term.
So think again about the amplitude.
but the three equations i got in post no 12 are they sufficient to find the solution?
is the amplitude 15cm?
but what is amplitude really here?
i mean not the value but the meaning of the term
in waves it is the up and down distance but here?
 
  • #16
Suyash Singh said:
what is amplitude really here?
In SHM the amplitude is the maximum displacement each side from a central equilibrium position.
Where is the equilibrium position in this case?
 
  • #17
If you draw a diagram of the motion of the mass you'll see the 15cm distance that the spring was stretched is greater than the distance from the equilibrium position to the fully stretched position. So the given distance x=15cm is greater than xm.

But you do know the distance x=15cm so you may be able to solve for xm with the mechanical energy at those two positions.
 
  • #18
haruspex said:
In SHM the amplitude is the maximum displacement each side from a central equilibrium position.
Where is the equilibrium position in this case?
Send BoBs said:
If you draw a diagram of the motion of the mass you'll see the 15cm distance that the spring was stretched is greater than the distance from the equilibrium position to the fully stretched position. So the given distance x=15cm is greater than xm.

But you do know the distance x=15cm so you may be able to solve for xm with the mechanical energy at those two positions.
force=kx
mg/k=x
x=200/2000
x=0.1m

equilibrium position is 0.1 m
 
  • #19
Ok, so what is the initial displacement from equilibrium (i.e., the amplitude)?
 
  • #20
haruspex said:
Ok, so what is the initial displacement from equilibrium (i.e., the amplitude)?
0.15m-0.1m=0.05m
amplitude=0.05m
 
  • #21
Suyash Singh said:
0.15m-0.1m=0.05m
amplitude=0.05m
Right, so can you now write the complete expression? Don't forget to specify what your x represents.
 
  • Like
Likes Suyash Singh
  • #22
haruspex said:
Right, so can you now write the complete expression? Don't forget to specify what your x represents.
x(t)=0.05sin(10t)
x is displacement from equilibrium
 
  • #23
Suyash Singh said:
x(t)=0.05sin(10t)
x is displacement from equilibrium
Does that give the right displacement at t=0?
 
  • #24
haruspex said:
Does that give the right displacement at t=0?
displacement at t=0 is -0.1 m
oh :(
x(t)=0.05sin(10t) - 0.1
 
  • #25
Suyash Singh said:
displacement at t=0 is -0.1 m
oh :(
x(t)=0.05sin(10t) - 0.1
No, now you do not have it symmetric about x=0. Look back at your post #12. There is something there you have forgotten.
And it is the displacement from equilibrium position that is of interest. What is the initial displacement?
 
  • #26
x(t)=Asin(wt+phi)
at t=0
displacement from equilibrium is 0.05m
so
0.05=0.05sin(phi)
thus phi=90 degrees
x(t)=0.05sin(10t+90)
x(t)=0.05cos(10t)
 
  • #27
Suyash Singh said:
x(t)=Asin(wt+phi)
at t=0
displacement from equilibrium is 0.05m
so
0.05=0.05sin(phi)
thus phi=90 degrees
x(t)=0.05sin(10t+90)
x(t)=0.05cos(10t)
Yes!
 
  • Like
Likes Suyash Singh

1. What is the formula for calculating the displacement of a suspended point mass in a spring SHM system?

The formula for calculating the displacement of a suspended point mass in a spring SHM system is: x = A cos(ωt + φ), where x is the displacement, A is the amplitude, ω is the angular frequency, and φ is the phase angle.

2. How do you determine the amplitude of a spring SHM system?

The amplitude of a spring SHM system can be determined by measuring the maximum displacement of the point mass from its equilibrium position. Alternatively, if the initial conditions of the system are known, the amplitude can be calculated using the formula: A = xmax - xeq, where xmax is the maximum displacement and xeq is the equilibrium position.

3. What is the relationship between the frequency and the period of a spring SHM system?

The frequency and period of a spring SHM system are inversely proportional. This means that as the frequency increases, the period decreases, and vice versa. The formula for calculating the period is: T = 1/f, where T is the period and f is the frequency.

4. How does the mass of the point mass affect the displacement in a spring SHM system?

The mass of the point mass does not affect the displacement in a spring SHM system. The displacement is solely determined by the amplitude, frequency, and phase angle of the system. However, the mass does affect the period of the system, with a larger mass resulting in a longer period.

5. Can the displacement of a spring SHM system ever be negative?

Yes, the displacement of a spring SHM system can be negative. This indicates that the point mass is located below the equilibrium position. However, it is important to note that the amplitude of the system is always positive, and the negative displacement is relative to the equilibrium position.

Similar threads

  • Introductory Physics Homework Help
Replies
20
Views
2K
  • Introductory Physics Homework Help
Replies
24
Views
1K
  • Introductory Physics Homework Help
Replies
1
Views
6K
  • Introductory Physics Homework Help
Replies
5
Views
2K
  • Introductory Physics Homework Help
Replies
3
Views
1K
  • Introductory Physics Homework Help
Replies
16
Views
399
  • Introductory Physics Homework Help
Replies
7
Views
2K
  • Introductory Physics Homework Help
Replies
3
Views
1K
  • Introductory Physics Homework Help
Replies
17
Views
373
  • Introductory Physics Homework Help
Replies
15
Views
3K
Back
Top